The ages of dogs and cats at an animal shelter are shown. Make a Venn diagram to show the number of animals that are dogs and are more than 8 years old.
Species|Age
dog|8
Cat|9
dog|9
cat|5
dog|12
cat|13
dog|9
cat|6
dog|8
dog|11
dog|5
cat|2

Answers

Answer 1

The number 5 represents the 5 dogs that are more than 8 years old, and the Number 2 represents the 2 cats that are more than 8 years old.

Based on the given data, we can create a Venn diagram to illustrate the number of animals that are dogs and are more than 8 years old.

Let's label two intersecting circles representing dogs and cats respectively. In the region where the circles overlap, we will place the animals that are both dogs and more than 8 years old.

First, let's count the number of dogs that are more than 8 years old. Based on the data, we have the following dogs that fit this criterion:

- Dog: 8 (not more than 8 years old)

- Dog: 9

- Dog: 12

- Dog: 9

- Dog: 11

So, there are a total of 5 dogs that are more than 8 years old.

Now, let's count the number of cats that are more than 8 years old. Based on the data, we have the following cats that fit this criterion:

- Cat: 9

- Cat: 13

So, there are a total of 2 cats that are more than 8 years old.

To create the Venn diagram, we will place the number 5 inside the region representing dogs, and the number 2 inside the region representing cats. The region where the circles overlap will be left empty since there are no animals that are both dogs and cats in this dataset.

The Venn diagram representing the number of animals that are dogs and are more than 8 years old would look as follows:

         Dogs

       ___________

      |      |      |

      |  5   |      |

      |______|______|

         Cats

       ___________

      |      |      |

      |      |  2   |

      |______|______|

In the Venn diagram, the number 5 represents the 5 dogs that are more than 8 years old, and the number 2 represents the 2 cats that are more than 8 years old.

To know more about Number .

https://brainly.com/question/26460978

#SPJ8

The Ages Of Dogs And Cats At An Animal Shelter Are Shown. Make A Venn Diagram To Show The Number Of Animals

Related Questions

The amount of time a certain brand of light bulb lasts is normally distributed with a mean of 2000 hours and a standard deviation of 25 hours. Out of 665 freshly installed light bulbs in a new large building, how many would be expected to last between 2030 hours and 2060 hours, to the nearest whole number?

Answers

To determine the number of light bulbs expected to last between 2030 hours and 2060 hours, we need to calculate the z-scores corresponding to these values and then use the z-score formula to find the proportion of light bulbs within this range.

The z-score formula is given by:

z = (x - μ) / σ

where:

x = value

μ = mean

σ = standard deviation

For 2030 hours:

z1 = (2030 - 2000) / 25

For 2060 hours:

z2 = (2060 - 2000) / 25

Now, we can use the z-scores to find the proportions associated with each value using a standard normal distribution table or calculator. The table or calculator will provide the area/proportion under the normal curve between the mean and each z-score.

Let's calculate the z-scores and find the proportions:

z1 = (2030 - 2000) / 25 = 1.2

z2 = (2060 - 2000) / 25 = 2.4

Using a standard normal distribution table or calculator, we can find the proportions corresponding to these z-scores:

P(z < 1.2) ≈ 0.8849

P(z < 2.4) ≈ 0.9918

To find the proportion of light bulbs expected to last between 2030 hours and 2060 hours, we subtract the cumulative probabilities:

P(2030 < x < 2060) = P(z1 < z < z2) = P(z < z2) - P(z < z1)

P(2030 < x < 2060) ≈ 0.9918 - 0.8849

Finally, we multiply this proportion by the total number of light bulbs (665) to get the estimated number of light bulbs expected to last between 2030 hours and 2060 hours:

Number of light bulbs ≈ (0.9918 - 0.8849) * 665

Rounding to the nearest whole number, the expected number of light bulbs that would last between 2030 hours and 2060 hours is approximately 71.[tex]\huge{\mathfrak{\colorbox{black}{\textcolor{lime}{I\:hope\:this\:helps\:!\:\:}}}}[/tex]

♥️ [tex]\large{\textcolor{red}{\underline{\mathcal{SUMIT\:\:ROY\:\:(:\:\:}}}}[/tex]

(q11) Find the center of mass of the system of objects that have masses 2 , 3 and 5 at the point (-1,2),(1,1) and (3,3) respectively.

Answers

The center of mass of the system is approximately (3.7, 2.6).

The center of mass of a system of objects is the point where all the weight of the system appears to be concentrated. It can be defined as the average location of the weighted parts of the system.

The center of mass of a system is dependent on the mass of the objects in the system and their positions.

Let's determine the center of mass of the system with masses of 2, 3, and 5 at the points (-1, 2), (1, 1), and (3, 3), respectively. Let's name the masses m1, m2, and m3, respectively, and the coordinates (x1, y1), (x2, y2), and (x3, y3).

The x-component of the center of mass is given by the formula:

x= (m1x1 + m2x2 + m3x3) / (m1 + m2 + m3)

The y-component of the center of mass is given by the formula:

y= (m1y1 + m2y2 + m3y3) / (m1 + m2 + m3)

By using the given values, let's calculate the x and y components of the center of mass:

x = (2 x -1 + 3 x 1 + 5 x 3) / (2 + 3 + 5) = 37/10 ≈ 3.7y

= (2 x 2 + 3 x 1 + 5 x 3) / (2 + 3 + 5)

= 26/10 = 2.6

To learn more about : mass

https://brainly.com/question/28916233

#SPJ8

3) Last year the mean salary for professors in a particular community college was $62,000 with a standard deviation of $2000. A new two year contract is negotiated. In the first year of the contract, each professor receives a $1500 raise.

Find the mean and standard deviation for the first year of the contract.
b) In the second year of the contract, each professor receives a 3% raise based on their salary during the first year of the contract. Find the mean and the standard deviation for the second year of the contract.

Answers

a) Mean for the first year of the contract: $63,500

The standard deviation for the first year of the contract: $2,000.

b) Mean for the second year of the contract: $65,405.

The standard deviation for the second year of the contract: $60.

We have,

To find the mean and standard deviation for the first year of the contract, we can use the given information and the properties of the normal distribution.

Given:

The mean salary for professors in the previous year = $62,000

Standard deviation in the previous year = $2,000

Raise in the first year = $1,500

Mean for the first year of the contract:

The mean salary for the first year can be obtained by adding the raise to the previous mean:

Mean = Previous Mean + Raise

Mean = $62,000 + $1,500

Mean = $63,500

The standard deviation for the first year of the contract:

Since each professor receives the same raise, the standard deviation remains the same:

Standard Deviation = $2,000

Therefore, for the first year of the contract, the mean salary is $63,500, and the standard deviation remains $2,000.

Now,

In the second year of the contract, each professor receives a 3% raise based on their salary during the first year of the contract.

To find the mean and standard deviation for the second year, we can use the given information and the properties of the normal distribution.

Mean for the second year of the contract:

To calculate the mean for the second year, we need to add a 3% raise to the mean salary of the first year:

Mean = Mean of the first year + (3% * Mean of the first year)

Mean = $63,500 + (0.03 * $63,500)

Mean = $63,500 + $1,905

Mean = $65,405

The standard deviation for the second year of the contract:

Since each professor receives a raise based on their salary from the first year, the standard deviation also increases. To calculate the standard deviation, we multiply the standard deviation from the first year by the percentage increase:

Standard Deviation = Standard Deviation of the first year * (Percentage Increase / 100)

Standard Deviation = $2,000 * (3 / 100)

Standard Deviation = $2,000 * 0.03

Standard Deviation = $60

Therefore, for the second year of the contract, the mean salary is $65,405, and the standard deviation is $60.

Thus,

a) Mean for the first year of the contract: $63,500

The standard deviation for the first year of the contract: $2,000.

b) Mean for the second year of the contract: $65,405.

The standard deviation for the second year of the contract: $60.

Learn more about mean here:

https://brainly.com/question/23263573

#SPJ1

Determine the a) total annual cost, and b) cost per mile to the nearest cent.
1. Liz Nolan drove 34,500 miles last year. The total of fixed costs was $9,916 and of variable costs was
$4,897.

Answers

Answer:

total annual cost: 49313

cost per mile: 14 cents

Step-by-step explanation:

find total annual cost by adding everything up

find cost per mile by doing 4897/34500

cost/ miles

we use variable cost since the only thing that might change each year is the amount of miles they drive

fixed costs are fixed and don't change

Find the amplitude of this function.
In
++
t
Give your answer as a decimal.

Answers

Answer:

2.5

Step-by-step explanation:

The explanation is attached below.

Find the measure of ∠F
.

Answers

Step-by-step explanation:

triangle EFG is an isosceles triangle

angle G

= 180°-58°

= 122° (adj. angles on a str. line)

angle F

= (180°-122°)÷2

= 29° (angles in a triangle)

The number of combinations of eight items taken three at a time can be written as

Answers

Answer: 8C3

Step-by-step explanation: You need to use Combinations for this. Out of 8, you need to select 3, so answer is 8C3.

Multiply three consecutive digits backwards starting from 8, and divide by 3 factorial

(8*7*6)/(3*2*1)

=56

what is the greatest common factor of 97 and 24? what the answer

Answers

1

Because the number 97 is a prime number

Answer:

The greatest common factor (GCF) of two numbers is the largest number that divides evenly into both numbers. Since 97 is a prime number and 24 is not divisible by 97, the GCF of 97 and 24 is 1.

please help! mathematicians

Answers

Answer:

1 < m < 4

Step-by-step explanation:

If the roots of function f(x) are not real, then the discriminant (the part under the square root sign) will be negative.

Set the discriminant less than zero and rewrite in standard form:

[tex]\begin{aligned}16-4m(-m+5)& < 0\\16+4m^2-20m& < 0\\4m^2-20m+16& < 0\\4(m^2-5m+4)& < 0\\m^2-5m+4& < 0\end{aligned}[/tex]

Factor the quadratic:

[tex]\begin{aligned}m^2-5m+4& < 0\\m^2-4m-m+4& < 0\\m(m-4)-1(m-4)& < 0\\(m-1)(m-4)& < 0\end{aligned}[/tex]

The leading coefficient of the quadratic m² - 5m + 4 is positive.

Therefore, the graph will be a parabola that opens upwards.

This means that the interval where the parabola is below the x-axis (negative) is between the zeros of the quadratic. Since the zeros are m = 1 and m = 4, the solution to the inequality is 1 < m < 4.

Therefore, the values of m for which the roots of function f(x) will be non-real are 1 < m < 4.

Jessica needs to know how much water her new fish tank can hold:

A rectangular prism with a length of 8 inches, a width of 4 inches, and a height of 9 inches.

Determine the total volume of the fish tank.

Answers

The fish tank has a total volume of 288 inch³. As a result, Jessica's new fish tank has a capacity of 288 inch³ for water.

The volume of a rectangular prism can be calculated using the formula:

V = l x b x h..........(i)

where,

V ⇒ Volume

l  ⇒ length

b ⇒ width

h ⇒ height

From the question, we are given the values,

l = 8 inches

b = 4 inches

h = 9 inches

Putting these values in equation (i), we get,

V = 8 x 4 x 9

⇒ V = 288 in³

Therefore, the fish tank has a total volume of 288 inch³. As a result, Jessica's new fish tank has a capacity of 288 inch³ for water.

Learn more about the volume of rectangular prism on:

https://brainly.com/question/24284033

vardan's homework assignment contains 24 problems of 58 1/3 of them are geometry. how many geometry problems are there?

Answers

There are 14 Geometry problems in Vardan's homework assignment.

The number of geometry problems in Vardan's homework assignment, we need to calculate 58 1/3 percent of the total number of problems.

First, let's convert 58 1/3 percent to a decimal by dividing it by 100:

58 1/3 percent = 58.33/100 = 0.5833

Next, we multiply the decimal by the total number of problems:

Number of geometry problems = 0.5833 * 24

To calculate this, we can multiply 0.5833 by 24:

Number of geometry problems = 0.5833 * 24 = 14

Therefore, there are 14 geometry problems in Vardan's homework assignment.

For more questions on Geometry .

https://brainly.com/question/31120908

#SPJ8

plssssssssssssssssssssssssssssssssssssssssssss answe in 5 mins

Answers

Answer:

Because we are adding 2/5, we would be moving in the positive direction, which is to the right.

A scientist mixes water (containing no salt) with a solution that contains 35% salt. She wants to obtain 140 ounces of a mixture that is 15% salt. How many
ounces of water and how many ounces of the 35% salt solution should she use?

Answers

Answer:

.35x = 140(.15)

.35x = 21

x = 60 oz of 35% salt.

The scientist will need 60 oz of the 35% salt solution and 80 oz of water.

How do you solve the question Deloitte signs a contract on December 1 to provide 40 days of advisory services with receipt of $20,000 due at the end of the contract. On December 31, 75% of the services have been completed.

Answers

As of December 31, Deloitte should recognize $15,000 as revenue for the advisory services completed.

To solve the given question, we need to determine the amount of revenue that Deloitte should recognize as of December 31, based on the percentage of services completed.

Here's how we can calculate it:

Calculate the total revenue for the contract:

Total revenue = $20,000

Determine the percentage of services completed:

Percentage of services completed = 75%

Calculate the revenue recognized as of December 31:

Revenue recognized = Percentage of services completed × Total revenue

= 75% × $20,000

= $15,000

Therefore, as of December 31, Deloitte should recognize $15,000 as revenue for the advisory services completed.

Learn more about revenue click;

https://brainly.com/question/29567732

#SPJ1

a is an arithmetic sequence where the 1st term of the sequence is {\textstyle\frac{3}{2}} and the 13th term of the sequence is -{\textstyle\frac{81}{2}}. Find the 13th partial sum of the sequence.

Answers

Answer:

195

Step-by-step explanation:

a = 3/2

According to the formula tn= a + (n-1)d

81/2= 3/2 + (13 - 1)d

81/2= 3/2 + 12d

81/3 = 12d

Therefore 27/12 = d

Sn= n/2 [2a + (n-1)d]

[tex]S_{13}[/tex] = 13/2 [2(3/2) + (13-1)(27/12)]

     = 13/2 (3 + 27)

     = 39/2 + 351/2

     = 390/2

     = 195

I NEED HELP WITH STATISTICS

Answers

(a) The null hypothesis is that the mean birth weight of babies born at full term is 7.2 pounds. The alternative hypothesis is that the mean birth weight of babies born at full term is greater than 7.2 pounds.

(b) If the scientist decides to reject the null hypothesis, she might be making a Type I error.

(c) A Type II error occurs when the null hypothesis is false, but the scientist fails to reject it.

How to explain the information

a A Type I error occurs when the null hypothesis is true, but the scientist rejects it. In this case, the null hypothesis is that the mean birth weight of babies born at full term is 7.2 pounds. If the scientist rejects this hypothesis, she is saying that she believes that the mean birth weight is greater than 7.2 pounds. However, if the null hypothesis is true, then the mean birth weight is actually 7.2 pounds, and the scientist has made a mistake.

b In this case, the scientist would fail to reject the null hypothesis and conclude that the mean birth weight of babies born at full term is 7.2 pounds. However, the true mean birth weight is 7.7 pounds, so the scientist would be making a Type II error.

c In the context of a Type II error, suppose the null hypothesis is false, meaning there is indeed a significant difference or relationship. However, due to various factors such as insufficient sample size, low statistical power, or other limitations, the scientist fails to reject the null hypothesis. Consequently, they accept the null hypothesis even though it is false, leading to a Type II error.

Learn more about hypothesis on

https://brainly.com/question/606806

#SPJ1

You spin the spinner once. 123 What is P(less than 2)? Write your answer as a fraction or whole number.

Answers

Answer:

See below

Step-by-step explanation:

Since the spinner has the numbers 1, 2, and 3 on it, and we want to find the probability of spinning a number less than 2, there is only one possible outcome that satisfies this condition, which is spinning a 1. Therefore, the probability of spinning a number less than 2 is:

P(less than 2) = P(1) = 1/3

So the probability of spinning a number less than 2 is 1/3.

prove that the points 2, -1+i√3, -1-i√3 for a equilateral triangle on the argand plane.
Find the length of a side of this trangle?

Answers

Answer:

The lengths are equal so the triangle is equilateral

Step-by-step explanation:

We can write the points as follows,

(2,0), (-1,[tex]\sqrt{3}[/tex]) (-1,-[tex]\sqrt{3}[/tex])

now if it is an equilateral triangle, all side lengths must be equal

first we compute the sides(vectors)

(2-(-1),-[tex]\sqrt{3}[/tex]) = (3,-[tex]\sqrt{3}[/tex]) = side 1

(2-(-1),[tex]\sqrt{3}[/tex]) = (3,[tex]\sqrt{3}[/tex]) = side 2

(-1+1,[tex]\sqrt{3}[/tex]+[tex]\sqrt{3}[/tex]) = (0,2[tex]\sqrt{3}[/tex]) = side 3

now we compute the lengths of the sides using pythagoras theorem

(3)^2 + (-[tex]\sqrt{3}[/tex])^2 = (length of side 1)^2 = 9 + 3 = 12

similarly, (3)^2 + ([tex]\sqrt{3}[/tex])^2 = 12 = Length of side 2 squared

and,( 2[tex]\sqrt{3}[/tex])^2 = length of side 3 squared = 12

since the squares are equal, so the lengths must also be equal

so the triangle is equilateral

this is just a quick addition to the superb posting by "hamza0100" above

well, indeed, in the argand or imaginary plane, for those values above we have the coordinates of A(2 , 0) , B(-1 √3) and C(-1 , -√3), let' use the distance formula for those fellows

[tex]~\hfill \stackrel{\textit{\large distance between 2 points}}{d = \sqrt{( x_2- x_1)^2 + ( y_2- y_1)^2}}~\hfill~ \\\\[-0.35em] ~\dotfill\\\\ A(\stackrel{x_1}{2}~,~\stackrel{y_1}{0})\qquad B(\stackrel{x_2}{-1}~,~\stackrel{y_2}{\sqrt{3}}) ~\hfill AB=\sqrt{(~~ -1- 2~~)^2 + (~~ \sqrt{3}- 0~~)^2} \\\\\\ ~\hfill AB=\sqrt{( -3)^2 + ( \sqrt{3})^2} \implies \boxed{AB=\sqrt{ 12 }}[/tex]

[tex]B(\stackrel{x_1}{-1}~,~\stackrel{y_1}{\sqrt{3}})\qquad C(\stackrel{x_2}{-1}~,~\stackrel{y_2}{-\sqrt{3}}) \\\\\\ BC=\sqrt{(~~ -1- (-1)~~)^2 + (~~ -\sqrt{3}- \sqrt{3}~~)^2} \\\\\\ ~\hfill BC=\sqrt{( 0)^2 + ( -2\sqrt{3})^2} \implies \boxed{BC=\sqrt{ 12 }}[/tex]

[tex]C(\stackrel{x_1}{-1}~,~\stackrel{y_1}{-\sqrt{3}})\qquad A(\stackrel{x_2}{2}~,~\stackrel{y_2}{0}) ~\hfill CA=\sqrt{(~~ 2- (-1)~~)^2 + (~~ 0- (-\sqrt{3})~~)^2} \\\\\\ ~\hfill CA=\sqrt{( 3)^2 + (-\sqrt{3})^2} \implies \boxed{CA=\sqrt{ 12 }} \\\\[-0.35em] ~\dotfill\\\\ AB=BC=CA=\sqrt{12}\implies 2\sqrt{3}\hspace{5em}\qquad equilateral\textit{\LARGE \checkmark}[/tex]

Que número estoy pensando si al multiplicarlo por 4 y luego de sumarle 16 obtengo 8?

Answers

Answer:-2

Step-by-step explanation:

x(4)+16=8

I NEED HELP WITH STATISTICS

Answers

Am here for you so need anything don’t message me

I need the solution!!!!​

Answers

Solve for the first variable in one of the equations, then substitute the result into the other equation.

Point form :
(-4,0)

Equation form :
x = -4, y = 0

22% of what number is 3300

Answers

To find the number that corresponds to 22% of a given value, you can divide the given value by 22% (or 0.22).

Let's use this approach to find the number:

3300 ÷ 0.22 = 15,000

So, 22% of 15,000 is equal to 3300.

Answer:

x = 15000

Step-by-step explanation:

If you are using a calculator, simply enter 3300×100÷22, which will give you the answer.

Two homebuyers are financing $137,000 to purchase a condominium. They obtained a 15-year, fixed-rate loan with a rate of 5.05%. They have been given the option of purchasing up to four points to lower their rate to 4.81%. How much will the four points cost them?

$1,370
$1,730
$4,580
$5,480

Answers

The cost of four points is:4 x $1,370 = $5,480Thus, the four points will cost the homebuyers $5,480.

Points can help lower mortgage rates on fixed-rate loans. The concept of points, which are basically prepaid interest, is a little complicated.

Each point is worth one percent of the loan amount, and paying points can lower your interest rate by a certain amount, typically about one-eighth to one-quarter of a percentage point.

The cost of points in the given scenario can be found using the following steps:

The loan amount to purchase a condominium is $137,000. The homebuyers obtained a 15-year fixed-rate loan with a rate of 5.05%.

If the homebuyers opt for four points, their loan rate will decrease to 4.81%.

To figure out how much the points will cost the homebuyers, we must first determine the cost of one point. Since one point is equal to 1% of the loan amount, one point on a $137,000 loan is:1% of $137,000 = $1,370

To learn more about : cost

https://brainly.com/question/2292799

#SPJ8

QUESTION 1 1.1 1.2 1.4 Use the definition of the derivative (first principles) to determine f'(x) if f(x)=2x 1.3 Determine f'(x) from first principles if f(x)=9-x². Determine f'(x) from first principles if f(x)=-4x².​

Answers

Based on the functions given, it should be noted that the values will be 2, -2x and -8x.

How to calculate the value

Using the definition of the derivative, we have:

f'(x) = lim(h->0) [f(x + h) - f(x)] / h

= lim(h->0) [2(x + h) - 2x] / h

= lim(h->0) 2h / h

= lim(h->0) 2

= 2

Therefore, f'(x) = 2.

For f(x) = 9 - x²:

Using the definition of the derivative, we have:

f'(x) = lim(h->0) [f(x + h) - f(x)] / h

= lim(h->0) [9 - (x + h)² - (9 - x²)] / h

= lim(h->0) [9 - (x² + 2xh + h²) - 9 + x²] / h

= lim(h->0) [-2xh - h²] / h

= lim(h->0) (-2x - h)

= -2x

Therefore, f'(x) = -2x.

For f(x) = -4x²:

Using the definition of the derivative, we have:

f'(x) = lim(h->0) [f(x + h) - f(x)] / h

= lim(h->0) [-4(x + h)² - (-4x²)] / h

= lim(h->0) [-4(x² + 2xh + h²) + 4x²] / h

= lim(h->0) [-4x² - 8xh - 4h² + 4x²] / h

= lim(h->0) [-8xh - 4h²] / h

= lim(h->0) (-8x - 4h)

= -8x

Therefore, f'(x) = -8x.

Learn more about functions on

https://brainly.com/question/31878183

#SPJ1

Suppose there are 17 jelly beans in a box-2 red, 3 blue, 4 white, and 8 green. What part of the jelly beans is blue? As a decimal rounded to the nearest ten-thousandth (four decimal places)

Answers

Blue Jelly beans are 0.1764 part of total .

Given,

Total beans = 17

Blue = 3

Red =2

White =4

Green =8

Now,

Out of total , green jelly beans = 8/17

Out of total , red jelly beans = 2/17

Out of total , white jelly beans = 4/17

Out of total , blue jelly beans = 3/17

Hence the blue jelly beans are 0.1764 part of total jelly beans .

Know more about decimal,

https://brainly.com/question/8985071

#SPJ1

Express 75 as a product of its prime factors write the prime factors in ascending order and give your answer in index form

Answers

Step-by-step explanation:

75 = 3 x 5 x 5    in prime factorization

Answer:

Step-by-step explanation:

3x5x5

546, 400 and 4,856 The value of 4 in which number is how many times larger than the value of 4 in which number.​

Answers

To determine how many times larger the value of 4 is in the second number compared to the first number, we need to calculate the ratio of the values.

First number: 546
Second number: 4,856

In the first number, the value of 4 is the same as the digit itself since it appears once.

In the second number, the value of 4 is larger since it appears twice.

To find the ratio, we divide the value of 4 in the second number by the value of 4 in the first number:

Value of 4 in second number: 2
Value of 4 in first number: 1

Ratio: 2/1 = 2

Therefore, the value of 4 in the second number is two times larger than the value of 4 in the first number.

Problem
Find the equation of the line.
Use exact numbers.

Answers

The Equation of line is y= -3/2x + 60

From the graph we take two coordinates as (2, 0) and (0, 3)

We know the formula for slope

Slope= (Change in y)/ (Change in x)

Slope = (3-0)/ (0-2)

Slope= 3 / (-2)

Slope= -3/2

Now, Equation of line

y - 0 = -3/2 (x-  2)

y= -3/2x + 6

Thus, the Equation of line is y= -3/2x + 60.

Learn more about Slope here:

https://brainly.com/question/3605446

#SPJ1

Which is the equation of the given line in point-slope form?

y−0=−1(x−8)

y−0=1(x+8)

y=−x+8

y−8=−1(x+0)

Answers

Answer:

y = -x + 8

Step-by-step explanation:

Let's break down the equation step by step to understand it better.

The equation in point-slope form is given as:

y - y1 = m(x - x1)

In this case, we have:

y - 0 = -1(x - 8)

The point-slope form uses a specific point (x1, y1) on the line and the slope (m) of the line.

Here, the point (x1, y1) is (8, 0), which represents a point on the line. This means that when x = 8, y = 0. The graph has a point at (8, 0), which confirms this information.

The slope (m) is -1 in this equation. The slope represents the rate at which y changes with respect to x. In this case, since the slope is -1, it means that for every unit increase in x, y decreases by 1. The negative sign indicates that the line has a downward slope.

By substituting the values into the equation, we get:

y - 0 = -1(x - 8)

Simplifying further:

y = -x + 8

This is the final equation of the line in slope-intercept form. It tells us that y is equal to -x plus 8. In other words, the line decreases by 1 unit in the y-direction for every 1 unit increase in the x-direction, and it intersects the y-axis at the point (0, 8).

If the graph has points at (0, 8) and (8, 0), the equation y = -x + 8 accurately represents that line.

A number divided by 10 is less than 4

Answers

2 is the correct answer

Answer: 2

Step-by-step explanation: 10 divided by 5 equals 2

Other Questions
Observing someone else succeed at challenging activities tends to make people feel insecure and disempowered. As a result, individuals should be sheltered from overachievers.true or false Required information [The following information applies to the questions displayed below.] Project Y requires a $340,500 investment for new machinery with a four-year life and no salvage value. The project yields the following annual results. Cash flows occur evenly within each year. (PV of $1. FV of $1. PVA of $1. and FVA of $.1) (Use appropriate factor(s) from the tables provided.) Annual Amounts Project YSales of new product $375,000Expenses Materials, labor, and overhead (except depreciation) 168,000 Depreciation-Machinery 85,125 Selling, general, and administrative expenses 27,000Income $94,8753. Compute Project Y's accounting rate of return. Jefferson qualifies for an income-adjusted monthly payment of $435. If Jefferson has a subsidized student loan of $44,000 at an annual interest rate of 4% (compounded monthly), how many months are required to repay the loan? (Round your answer up to the nearest month.) dynamic nature of monopolistic competition and discuss some approaches they have used (or are currently using) to compete and survive in the market.In addition, provide an example or a scenario from your personal (or professional) experience, an observation, a story that you have read, or an idea or a thought that you might have for practical, creative, and/or effective ways to minimize production costs in order to maximize profits. Provide your opinion of the importance of the writing skills ofthe students in the business writing class in one (1) short welldeveloped paragraph.write a short paragraph Find the largest t-interval on which the existence-uniqueness theorem guarantees a unique solution for the following the initial problem. y' - ty/t + 4 = e^t/sin t, y(- pi/2) = -1 (t - 1)y' - ln (5 - t)/t - 3, y(2) = 4 Determine whether the discrete probability distribution is valid. a) Is this a valid discrete probability distribution: [Select] No Yes X P(X) 1 0.34 0.12 3 0.41 0.65 0.02 b) Is this a valid discre How long does it take for $14050 to grow to $26500, if interest rates are set at 15%? O 4.54 years O 423.33 years O 0.59 years 12.23 years QUESTION 15 How long does it take for $14050 to grow to $26500, if interest rates are set at 15%? O 4.54 years O 423.33 years O 0.59 years 12.23 years QUESTION 15 How long does it take for $14050 to grow to $26500, if interest rates are set at 15%? O 4.54 years O 423.33 years O 0.59 years 12.23 years listen to exam instructions you are currently using a cellular network plan that provides maximum speeds of up to 10 gbps for your smart phone. which of the following cellular network types are you using? a 30-year-old woman presents with 3 days of generalized weakness, dizziness, and excessive urination. she is conscious but restless, and she tells you that she is extremely thirsty. her blood pressure is 96/66 mm hg, her pulse is 110 beats/min and full, and her respirations are rapid and deep. on the basis of this patient's clinical presentation, she will most likely require: Horn, Inc. stock has a beta of 1.1 and Murray, Inc. stock has a beta of 0.7. According to the CAPM, which of the following statements is most accurate?Group of answer choicesA) The alpha of the Murray stock should be greater than the alpha of Horn stock.B) The stock of Horn, Inc. has more systematic risk than that of Murray, Inc.C) The expected return should be higher for the stock of Murray, Inc.D) The stock of Horn, Inc. has more total risk than Murray, Inc. Which of the following are assumptions that need to be met when conducting a one-sample t- test? Check all that apply. th Sample size is below 30 ,The population standard deviation is known, Observations are independent of each other ,Population is normally distributed The table shows the outcome of car accidents in a certain state for a recent year by whether or not the driver wore a seat belt. No Seat Belt Wore Seat Belt 412.777 163,916 Driver Survived Driver Died 507 413,284 2354 166,270 Total Find the probability of wearing seat belt, given that the driver survived a car accident. The probability as a decimal is (Round to three decimal places as needed.) Total 576,693 2861 579,554 Peter and Manuela met in their high school senior year psychology class. They sat near each other and were often partnered during class discussions and group work. They were both on the swim team and often volunteered at the same homelessness prevention shelter. By the end of the year, they became good friends. Later, Manuela and Peter attended the same college, and after graduation they became engaged. When they attended their five-year high school reunion it was obvious to their friends from high school that they were a very happy couple. Explain the reward theory of attraction and give an example to show how each of the following factors may have influenced Manuela and Peters developing relationship from high school through college. -Proximity. -Equity. -Self-disclosure. - Macroland produces dishes and glassware. Before trade, a set of dishes sells for $100 and a set of glasses sells for $50. When Macroland opens to trade, foreign demand for domestically produced china is strong, raising the price of a set of dishes to $125. But foreign competition reduces the demand for domestically produced glasses, so they now sell for $25 a set. Assuming workers cannot move between industries, the wages of workers producing dishes will___ and the wages of workers producing glasses will ___1. increase; not change 2. decrease; increase 3. increase; decrease 4. increase; increase A sculptor has prepared a mold for casting a bronze figure. The figure has a volume of 225 mL. If the bronze has a density of 7.8 g/mL, how many ounces of bronze are needed in the preparation of the bronze figure? Lins father is paying for a 40.00 meal. 7% states tax applied and he wants to leave a 10% tip. What does lins father pay for the meal? MM company is considering investing in Project 1 or Project 2. Project 1 generates the following cash flows: year "zero" = 321 dollars (outflow); year 1 = 169 dollars (inflow); year 2 = 318 dollars (inflow); year 3 = 338 dollars (inflow); year 4 = 152 dollars (inflow). Project 2 generates the following cash flows: year "zero" = 410 dollars (outflow); year 1 = 130 dollars (inflow); year 2 = 100 dollars (inflow); year 3 = 190 dollars (inflow); year 4 = 120 dollars (inflow). The MARR is 10 %. Using the Present Worth Method, calculate the Net Present Value of the BEST project. MFAIZ Holdings Inc. is a retailer of kitchen accessories and hardware. The company is estimating its cash requirement for upcoming months. The firms history and predictions of sales in 2021 and 2022 are as follows:Additional information:(i) The company is adopting an old sales policy whereby 55 percent of sales on cash basis. Another 30 percent is to be collected in 1 month after the sales while the remaining 15 percent is to be collected within 2 months after the sales.(ii) The company purchases 60 percent of materials two months before the anticipated sales.(iii) Payment of purchases are 40 percent, which is paid in 1 month following the purchases, and the remaining is to be paid in the next 2 months after purchases.(iv) The company has estimated RM3,000 monthly for their fixed operating cost.(v) The monthly wages and salaries are 10 percent of the monthly sales.(vi) Rental of premises is RM5,000 per month respectively.(vii) EPF contributions RM12,000 for February and March 2022.(viii) The companys initial cash balance for the month of January is RM12,000 and the company has a policy of maintaining a minimum cash requirement of RM15,000.Based on the information provided, prepare a cash budget for the first quarter of the 2022. How would you rewrite the following JavaScript function using arrow function syntax? function totalWithTax(price1, price2) { return (price1 + price2) * 1.09; } Question options: A totalWithTax(price1 + price2) => (price1 + price2) * 1.09; B totalWithTax => (price1 + price2) * 1.09; C let totalWithTax = => (price1 + price2) * 1.09; D let totalWithTax = (price1, price2) => (price1 + price2) * 1.09;